Question

Problem 20.71 A long, straight wire carries a current of 10.0 A, as shown in the figure 5.00 cm A ←-7.00 cm 10.0A Part A Use the law of Blot and Savart to find the direction of the magnetic field at point P due to the 2.20 mm segment A ANSWER The field is directed into the page. The field is directed out of the page. The field is directed to the right. The field is directed to the left. Part B Use the law of Biot and Savart to find the magnitude of the magnetic field at point P due to the 2.20 mm segment A ANSWER: BA = Use the law of Biot and Savart to find the direction of the magnetic field at point P due to the 2.20 mm segment C. ANSWER:

0 0
Add a comment Improve this question Transcribed image text
Answer #1

Here ,

Biot savart's law is

B = u0*I*dl X r/(4pi*r^3)

part A)

the direction of field due to sagment A is Out of the page

the field is out of the page ,

part B )

for point A , r = sqrt(7^2 + 5^2)

r = 8.60 cm = 0.086 m

field at A = u0 * ( 10 * 2.2 *10^-3 * 0.086)/(4pi* 0.086^3)

BA = 2.98 *10^-6 T

the magnetic field due to sagment A is 2.98 *10^-6 T

part C )

for sagment C ,

r = 0.05 m

the direction of field due to sagment C is Out of the page

the field is out of the page ,

part D)


field at D = u0 * ( 10 * 2.2 *10^-3 * 0.05)/(4pi* 0.05^3)

BD = 8.802 *10^-6 T

the magnetic field due to sagment A is 8.802 *10^-6 T

Add a comment
Know the answer?
Add Answer to:
Problem 20.71 A long, straight wire carries a current of 10.0 A, as shown in the...
Your Answer:

Post as a guest

Your Name:

What's your source?

Earn Coins

Coins can be redeemed for fabulous gifts.

Not the answer you're looking for? Ask your own homework help question. Our experts will answer your question WITHIN MINUTES for Free.
Similar Homework Help Questions
  • The diagram below shows a long wire with a right angle bend. A current of -3.25...

    The diagram below shows a long wire with a right angle bend. A current of -3.25 A flows through the wire. Determine the magnetic field in unit vector notation) at the point which is a distance - 2.12 cm from the corner of the bend. (Assume that the+xaxis is directed to the right, they axis is directed up, and the +7 xs is toward you.) B-153.77 Can you find an expression for the result of applying the Biot Savat law...

  • Circular Loops - CL-1 A circular wire of radius R carries a current I as shown to the right. dl2 to determine a) Use the Biot-Savart law dB the direction of dB1, the magnetic field produced at the...

    Circular Loops - CL-1 A circular wire of radius R carries a current I as shown to the right. dl2 to determine a) Use the Biot-Savart law dB the direction of dB1, the magnetic field produced at the origin by ring element dli shown on the figure. Draw a figure like the one given and show on your figure dh and the unit vector State the direction of dB1 in terms of the coordinate axes. b) Use the Biot-Savart law...

  • Р [12]35. A piece of wire is bent in the shape shown (two 32.6 cm straight...

    Р [12]35. A piece of wire is bent in the shape shown (two 32.6 cm straight sections and a 3/4 of a circle section with radius r - 320 cm). Based on this information 17.4 m which of the following is true? Note that point P is the center of the 3/4th circular section. Circle the TRUE statement(s). Statements: (a) The magnetic field at point P is due only to the current in the two straight sections. (b) The law...

  • A very long wire carrying current I is bent into the shape shown in the diagram....

    A very long wire carrying current I is bent into the shape shown in the diagram. Apply the general result obtained in class from the Biot-Savart Law, B = mu_0I/4pi r(cos theta_1 - cos theta_2), to find the net magnetic field B, magnitude and direction, at point P. Show all your work in detail: very specifically show what are theta_1 and theta_2 for each segment of the wire. Simplify the answer. Use extra pages if needed.

  • A straight wire carries a 10.0-A current. ABCD is a rectangle with point D in the middle of a 1.10-mm segment of the w...

    A straight wire carries a 10.0-A current. ABCD is a rectangle with point D in the middle of a 1.10-mm segment of the wire and point C in the wire. Find the magnitude and direction of the magnetic field due to this segment at (a) point A; (b) point B; (c) point C.

  • Q4. (25 points) A long straight wire carries a current of 11 = 8 A is...

    Q4. (25 points) A long straight wire carries a current of 11 = 8 A is at perpendicular distance d = 6 cm from a point P. The point P is at the center of a semicircle of radius R = 12 cm, and carries a current 12 = 17 A in the clockwise direction. a) (8 points) Find the magnetic field vector at point P due to the current 11 in the wire. b) (8 points) Find the magnetic...

  • Consider the current-carrying wire shown in the figure. The current creates a magnetic field at the...

    Consider the current-carrying wire shown in the figure. The current creates a magnetic field at the point P, which is the center of the arc segment of the wire. If 0 = 20.0°, the radius of the arc is 0.700 m, and the current is 2.00 A, what are the magnitude (in nT) and direction of the field produced at P? P i magnitude 285.7 Apply the Biot-Savart law. What will be the contributions of the segments of wire along...

  • An infinitely long conductor carrying current is bent at a right angle as shown in Figure...

    An infinitely long conductor carrying current is bent at a right angle as shown in Figure 1. Point Pis located a distance b from the corner of the wire. Only one section of this current contributes to the magnetic field at pt. P. Why? The general formula (derived from the Biot-Savart Law) for the magnitude of the magnetic field a distance a away from a thin, straight conductor is: B = f (sin 8, - sin 02) For this problem,...

  • biot savart law Uu mportant: Always write all equations that are used to solve a problem....

    biot savart law Uu mportant: Always write all equations that are used to solve a problem. Show clearly what are the steps of the solving process and indicate the final answer. 1. A segment of a circular loop of wire carries a current I. which enters and leaves on straight segments of wire, which are along magnetic field at point C, assuming the angle between the straight segments equal to 72 degrees. a radial direction from the center of the...

  • Constants Part A A straight, nonconducting plastic wire 7.00 cm long carries a charge density of...

    Constants Part A A straight, nonconducting plastic wire 7.00 cm long carries a charge density of 125 nC/m distributed uniformly along its length. It is lying on a horizontal tabletop. Find the magnitude and direction of the electric field this wire produces at a point 5.00 cm directly above its midpoint. E: N/C Submit Request Answer Part B O electric field is directed upward O electric field is directed downward Submit Request Answer We were unable to transcribe this image

ADVERTISEMENT
Free Homework Help App
Download From Google Play
Scan Your Homework
to Get Instant Free Answers
Need Online Homework Help?
Ask a Question
Get Answers For Free
Most questions answered within 3 hours.
ADVERTISEMENT
ADVERTISEMENT
ADVERTISEMENT